-
Notifications
You must be signed in to change notification settings - Fork 0
/
Copy pathchapter11.tex
216 lines (195 loc) · 9.43 KB
/
chapter11.tex
1
2
3
4
5
6
7
8
9
10
11
12
13
14
15
16
17
18
19
20
21
22
23
24
25
26
27
28
29
30
31
32
33
34
35
36
37
38
39
40
41
42
43
44
45
46
47
48
49
50
51
52
53
54
55
56
57
58
59
60
61
62
63
64
65
66
67
68
69
70
71
72
73
74
75
76
77
78
79
80
81
82
83
84
85
86
87
88
89
90
91
92
93
94
95
96
97
98
99
100
101
102
103
104
105
106
107
108
109
110
111
112
113
114
115
116
117
118
119
120
121
122
123
124
125
126
127
128
129
130
131
132
133
134
135
136
137
138
139
140
141
142
143
144
145
146
147
148
149
150
151
152
153
154
155
156
157
158
159
160
161
162
163
164
165
166
167
168
169
170
171
172
173
174
175
176
177
178
179
180
181
182
183
184
185
186
187
188
189
190
191
192
193
194
195
196
197
198
199
200
201
202
203
204
205
206
207
208
209
210
211
212
213
214
215
216
%%%%
%%
%%
%%%%
%%%% CHAPTER 11
%%%% CHAPTER 11
%%%%
%%
%%
%%%%
\section{Chapter 11 Solutions}
\begin{center}\hyperref[toc]{\^{}\^{}}\end{center}
\begin{center}\begin{tabular}{lllllllllllllllllllllllll}
\hyperref[problem1chapter11]{P1} & \hyperref[problem2chapter11]{P2} & \hyperref[problem3chapter11]{P3} & \hyperref[problem4chapter11]{P4} & \hyperref[problem5chapter11]{P5} & \hyperref[problem6chapter11]{P6} & \hyperref[problem7chapter11]{P7}
\end{tabular}\end{center}
\setcounter{problem}{0}
\setcounter{solution}{0}
\begin{problem}\label{problem1chapter11}
Use the fact that
$$\exp(2xt-t^2) = \exp(2xt-x^2t^2) \exp[t^2(x^2+1)]$$
to obtain the expansion
$$H_n(x) = \displaystyle\sum_{k=0}^{[\frac{n}{2}]} \dfrac{n! H_{n-2k}(1) x^{n-2k}(x^2+1)^k}{k!(n-2k)!}.$$
\end{problem}
\begin{solution}
From
$$\exp(2xt-t^2) = \exp(2xt-x^2t^2)\exp[t^2(x^2+1)]$$
we obtain
\begin{eqnarray*}
\lefteqn{\left( \displaystyle\sum_{n=0}^{\infty} \dfrac{H_n(1) (xt)^n}{n!} \right) \left( \displaystyle\sum_{n=0}^{\infty} \dfrac{(x^2+1)^n t^{2n}}{n!} \right)} \\
&&= \displaystyle\sum_{n=0}^{\infty} \displaystyle\sum_{k=0}^{[\frac{n}{2}]} \dfrac{H_{n-2k}(1) (x)^{n-2k}(x^2+1)^k t^n}{k! (n-2k)!}.
\end{eqnarray*}
It follows that
$$H_n(x) = \displaystyle\sum_{k=0}^{[\frac{n}{2}]} \dfrac{n! H_{n-2k}(1) x^{n-2k} (x^2+1)^k}{k! (n-2k)!}.$$
\end{solution}
%%%%
%%
%%
%%%%
\begin{problem}\label{problem2chapter11}
Use the expansion of $x^n$ in a series of Hermite polynomials to show that
$$\displaystyle\int_{-\infty}^{\infty} \exp(-x^2)x^n H_{n-2k}(x) \mathrm{d}x = 2^{-2k}n! \dfrac{\sqrt{\pi}}{k!}.$$
Note in particular the special case $k=0$.
\end{problem}
\begin{solution}
We know that
$$x^n = \displaystyle\sum_{s=0}^{[\frac{n}{2}]} \dfrac{n! H_{n-2s}(x)}{2^n s! (n-2s)!}.$$
Then
\begin{eqnarray*}
\lefteqn{\displaystyle\int_{-\infty}^{\infty} \exp(-x^2) x^n H_{n-2k}(x) \mathrm{d}x} \\
&&= \displaystyle\sum_{s=0}^{[\frac{n}{2}]} \dfrac{n!}{^n s! (n-2k)!} \displaystyle\int_{-\infty}^{\infty} e^{-x^2} H_{n-2k}(x) H_{n-2s}(x) \mathrm{d}x.
\end{eqnarray*}
The integrals involved on the right are zero except for $s=k$. Hence
$$\begin{array}{ll}
\displaystyle\int_{-\infty}^{\infty} \exp(-x^2)x^n H_{n-2k}(x)\mathrm{d}x &= \dfrac{n!}{2^n k! (n-2k)!} \displaystyle\int_{\infty}^{\infty} e^{-x^2} H_{n-2k}^2(x) \mathrm{d}x \\
&= \dfrac{n!2^{n-2k}(n-2k)! \sqrt{\pi}}{2^n k! (n-2k)!} \\
&= \dfrac{n! \sqrt{\pi}}{2^{2k}k!}.
\end{array}$$
For $k=0$ the right member becomes $n! \sqrt{\pi}$.
\end{solution}
%%%%
%%
%%
%%%%
\begin{problem}\label{problem3chapter11}
Use the integral evaluation in equation (4), page 192, to obtain the result
$$\displaystyle\int_0^{\infty} \exp(-x^2) H_{2k}(x) H_{2n+1}(x) \mathrm{d}x = \dfrac{(-1)^{k+s}2^{2k+2s}(\frac{1}{2})_k (\frac{3}{2})_s}{2s+1-2k}.$$
\end{problem}
\begin{solution}
We know that for $m \neq n$,
$$\displaystyle\int_{-\infty}^{\infty} e^{-x^2} H_n(x) H_m(x) \mathrm{d}x =0.$$
If $m$ and $n$ are both odd or both even, the integrand above is an even function of $x$ and we obtain
$$2 \displaystyle\int_0^{\infty} e^{-x^2} H_n(x) H_m(x)\mathrm{d}x =0$$
for $m \equiv n \hspace{4pt}\mathrm{mod} \hspace{3pt} 2$.
Now consider
$$\displaystyle\int_0^{\infty} e^{-x^2} H_{2n}(x) H_{2s+1}(x)\mathrm{d}x.$$
By equation (4), page 331, we get
\begin{eqnarray*}
\lefteqn{2(2n-2s-1) \displaystyle\int_0^{\infty} e^{-x^2} H_{2n}(x) H_{2s+1}(x) \mathrm{d}x} \\
& &= \left[ e^{-x^2} \left\{ H_{2n}(x) H_{2s+1}'(x) - 2_{2n}'(x) H_{2s+1}(x) \right\} \right]_0^{\infty} \\
&&= -H_{2n}(0)H_{2s+1}'(0) + H_{2n}'(0)H_{2s+1}(0) \\
&&= -H_{2n}(0)H_{2s+1}'(0).
\end{eqnarray*}
Using the values of $H_{2n}(0)$ and $H_{2s+1}'(0)$ from page 323, we get
$$\begin{array}{ll}
\displaystyle\int_0^{\infty} e^{-x^2}H_{2n}(x)H_{2s+1}(x) \mathrm{d}x &= \dfrac{-(-1)^n2^{2n}(\frac{1}{2})_n (-1)^s 2^{2s+1} (\frac{3}{2})_s}{2(2n-2s-1)} \\
&= \dfrac{(-1)^{n+s} 2^{2n+2s} (\frac{1}{2})_n (\frac{3}{2})_s}{2s+1-2n}.
\end{array}$$
\end{solution}
%%%%
%%
%%
%%%%
\begin{problem}\label{problem4chapter11}
By evaluating the integral on the right, using equation (2), page 187, ans term-by-term integration, show that
$$(A) \hspace{30pt} P_n(x) = \dfrac{2}{n! \sqrt{\pi}} \displaystyle\int_0^{\infty} \exp(-t^2) t^n H_n(xt) \mathrm{d}t,$$
which is Curzon's integral for $P_n(x)$, equation (4), page 191.
\end{problem}
\begin{solution}
Consider the integral
$$\begin{array}{ll}
\dfrac{2}{n!\sqrt{\pi}} \displaystyle\int_0^{\infty} e^{-t^2} t^n H_n(xt) \mathrm{d}t &= \dfrac{2}{\sqrt{\pi}} \displaystyle\int_0^{\infty} e^{-t^2} t^n \displaystyle\sum_{k=0}^{[\frac{n}{2}]} \dfrac{(-1)^k (2xt)^{n-2k}}{k! (n-2k)!} \mathrm{d}t \\
&= \displaystyle\sum_{k=0}^{[\frac{n}{2}]} \dfrac{(-1)^k (2k)^{n-2k}}{k! (n-2k)!} \dfrac{1}{\sqrt{\pi}} \displaystyle\int_0^{\infty} e^{-\beta} \beta^{n-k-\frac{1}{2}} \mathrm{d} \beta \\
&= \displaystyle\sum_{k=0}^{[\frac{n}{2}]} \dfrac{(-1)^k (2x)^{n-2k}}{k! (n-2k)!} \dfrac{\Gamma(n-k+\frac{1}{2})}{\Gamma(\frac{1}{2})} \\
&= \displaystyle\sum_{k=0}^{[\frac{n}{2}]} \dfrac{(-1)^k (\frac{1}{2})_{n-k} (2x)^{n-2k}}{k! (n-2k)!}.
\end{array}$$
Hence
$$\dfrac{2}{n! \sqrt{\pi}} \displaystyle\int_0^{\infty} e^{-t^2} t^n H_n(xt) \mathrm{d}t = P_n(x).$$
\end{solution}
%%%%
%%
%%
%%%%
\begin{problem}\label{problem5chapter11}
Let $v_n(x)$ denote the right member of equation $(A)$ of Exercise~\ref{problem4chapter11}. Prove $(A)$ by showing that
$$\displaystyle\sum_{n=0}^{\infty} v_n(x) y^n = (1-2xy+y^2)^{-\frac{1}{2}}.$$
\end{problem}
\begin{solution}
Put $v_n(x) = \dfrac{2}{n! \sqrt{\pi}} \displaystyle\int_0^{\infty} e^{-t^2} t^n H_n(xt) \mathrm{d}t.$
Then
$$\begin{array}{ll}
\displaystyle\sum_{n=0}^{\infty} v_n(x)y^n &= \dfrac{2}{\sqrt{\pi}} \displaystyle\int_0^{\infty} e^{-t^2} \displaystyle\sum_{n=0}^{\infty} \dfrac{t^n H_n(xt) t^n}{n!} \mathrm{d}t \\
&= \dfrac{2}{\sqrt{\pi}} \displaystyle\int_0^{\infty} e^{-t^2} e^{2(xt)yt-y^2t^2}\mathrm{d}t \\
&= \dfrac{2}{\sqrt{\pi}} \displaystyle\int_0^{\infty} e^{-t^2[1-2xy+y^2]} \mathrm{d}t.
\end{array}$$
Use $t \sqrt{1-2xy+y^2}=\beta$ to get
$$\begin{array}{ll}
\displaystyle\sum_{n=0}^{\infty} v_n(x) y^n &= \dfrac{2}{\sqrt{\pi}\sqrt{1-2xy+y^2}} \displaystyle\int_0^{\infty} e^{-\beta^2} \mathrm{d} \beta \\
&= (1-2xy+y^2)^{-\frac{1}{2}} \\
&= \displaystyle\sum_{n=0}^{\infty} P_n(x) y^n.
\end{array}$$
Hence $v_n(x) = P_n(x).$
\end{solution}
%%%%
%%
%%
%%%%
\begin{problem}\label{problem6chapter11}
Evaluate the integral on the right in
$$(B) \hspace{30pt} H_n(x) = 2^{n+1} \exp(x^2) \displaystyle\int_x^{\infty} \exp(-t^2) t^{n+1} P_n \left( \dfrac{x}{t} \right) \mathrm{d}t$$
by using
$$(2t)^n P_n \left( \dfrac{x}{t} \right) = \displaystyle\sum_{k=0}^{[\frac{n}{2}]} \dfrac{n! (x^2-t^2)^k (2x)^{n-2k}}{(k!)^2 (n-2k)!}$$
derived from equation (1), page 164, and term-by-term integration to prove the validity of $(B)$, which is equation (5), page 191.
\end{problem}
\begin{solution}
We know
$$P_n(x) = \displaystyle\sum_{k=0}^{[\frac{n}{2}]} \dfrac{n! (x^2-1)^k x^{n-2k}}{2^{2k}(k!)^2 (n-2k)!}$$
from (1), page 280. Then
$$(2t)^n P_n \left( \dfrac{x}{t} \right) = \displaystyle\sum_{k=0}^{[\frac{n}{2}]} \dfrac{n! (x^2-t^2)^k (2x)^{n-2k}}{(k!)^2 (n-2k)!}$$
which leads to the result
\begin{eqnarray*}
\lefteqn{2^{n+1} e^{x^2} \displaystyle\int_x^{\infty} e^{-t^2} t^{n+1} P_n \left( \dfrac{x}{t} \right) \mathrm{d}t} \\
&& = \displaystyle\sum_{k=0}^{[\frac{n}{2}]} \dfrac{n! e^{x^2}}{(k!)^2 (n-2k)!} \displaystyle\int_x^{\infty} e^{-t^2} (x^2-t^2)^k (2x)^{n-2k}(2t \mathrm{d}t).
\end{eqnarray*}
Put $t-x^2=\beta$. Then
\begin{eqnarray*}
\lefteqn{2^{n+1} e^{x^2} \displaystyle\int_x^{\infty} e^{-t^2} t^{n+1}P_n \left( \dfrac{x}{t} \right) \mathrm{d}t} \\
& &= \displaystyle\sum_{k=0}^{[\frac{n}{2}]} \dfrac{n! (2x)^{n-2k}}{(k!)^2 (n-2k)!} \displaystyle\int_0^{\infty} e^{-\beta} (-1)^k \beta^k \mathrm{d} \beta \\
& &= \displaystyle\sum_{k=0}^{[\frac{n}{2}]} \dfrac{n! (-1)^k (2x)^{n-2k} k!}{(k!)^2 (n-2k)!} \\
& &= \displaystyle\sum_{k=0}^{[\frac{n}{2}]} \dfrac{n! (-1)^k (2x)^{n-2k}}{k! (n-2k)!}
\end{eqnarray*}
Hence
$$2^{n+1} e^{x^2} \displaystyle\int_x^{\infty} e^{-t^2} t^{n+1} P_n \left( \dfrac{x}{t} \right) \mathrm{d}t = H_n(x).$$
\end{solution}
%%%%
%%
%%
%%%%
\begin{problem}\label{problem7chapter11}
Use the Rodrigues formula
$$\exp(-x^2)H_n(x) = (-1)^n D^n \exp(-x^2); \mathscr{D} := \dfrac{\mathrm{d}}{\mathrm{d}x}$$
an iteration integration by parts to show that
$$\displaystyle\int_{-\infty}^{\infty} \exp(-x^2)H_n(x) H_m(x) \mathrm{d}x = \left\{ \begin{array}{ll}
0 &; m \neq n \\
2^n n! \sqrt{\pi} &; m=n
\end{array} \right.$$
\end{problem}
\begin{solution}
We know $H_n(x) = (-1)^n e^{x^2} \mathscr{D}^n e^{-x^2}$.
Then
\begin{eqnarray*}
\lefteqn{\displaystyle\int_{-\infty}^{\infty} e^{-x^2} H_n(x) H_m(x) \mathrm{d}x} \\
& &= (-1)^n \displaystyle\int_{\infty}^{\infty} [\mathscr{D}^n e^{-x^2}] H_m(x) \mathrm{d}x \\
& &= (-1)^n \left[ \left\{ \mathscr{D}^{n-1} e^{-x^2} \right\} H_m(x) \right]_{\infty}^{\infty} + (-1)^{n+1} \displaystyle\int_{-\infty}^{\infty} [\mathscr{D}^{n-1}e^{-x^2}][\mathscr{D}H_m(x)]\mathrm{d}x \\
& &= \ldots \\
& &= (-1)^{2n} \displaystyle\int_{-\infty}^{\infty} e^{-x^2} [\mathscr{D}^n H_m(x)] \mathrm{d}x.
\end{eqnarray*}
Hence
$$\displaystyle\int_{-\infty}^{\infty} e^{-x^2} H_n(x) H_m(x) \mathrm{d}x = 0, m \neq n$$
and, since $H_n(x) = 2^n x^n + \pi_{n-2}(x)$ (where $\pi_{n-2}(x)$ denotes a polynomial of degree $(n-2)$),
$$\begin{array}{ll}
\displaystyle\int_{-\infty}^{\infty} e^{-x^2}H_n^2(x) \mathrm{d}x &= \displaystyle\int_{-\infty}^{\infty} e^{-x^2} [\mathscr{D}^n H_n(x)] \mathrm{d}x \\
&= 2^n n! \displaystyle\int_{-\infty}^{\infty} e^{-x^2} \mathrm{d}x \\
&= 2^n n! \sqrt{\pi}.
\end{array}$$
\end{solution}